ECE 280/Examples/Convolution

From PrattWiki
Revision as of 23:49, 16 September 2013 by DukeEgr93 (talk | contribs)
(diff) ← Older revision | Latest revision (diff) | Newer revision → (diff)
Jump to navigation Jump to search

The following is an example of convolving two signals; the convolution is done several different ways:

  1. Math... So much math.
  2. Using Convolution Shortcuts
  3. Geometrically, flipping and shifting \(h(t)\)
  4. Geometrically, flipping and shifting \(x(t)\)

Setup

The goal for this problem is to determine the output \(y(t)\) created by an input \(x(t)\) for a linear time invariant system if the system's impulse response \(h(t)\) is known. In this particular example:

\( \begin{align} x(t)&=2u(t)-u(t-1)-u(t-3)\\ h(t)&=e^{-t}u(t)\\ y(t)&=x(t)*h(t)=\int_{-\infty}^{\infty}x(\tau)h(t-\tau)d\tau=\int_{-\infty}^{\infty}x(t-\tau)h(\tau)d\tau \end{align} \)

where the particular choice of which integral to use is up to the user.

Math

Given the relative complexity of \(x(t)\), it may make more sense to use the first form, where \(h(t)\) is flipped and shifted. That yields:

\( \begin{align} y(t)&=\int_{-\infty}^{\infty}x(\tau)h(t-\tau)d\tau\\ y(t)&=\int_{-\infty}^{\infty}\left(2u(\tau)-u(\tau-1)-u(\tau-3)\right)\left(e^{-(t-\tau)}u(t-\tau)\right)d\tau \end{align} \)

Distributing terms gives:

\( \begin{align} y(t)&=\int_{-\infty}^{\infty}2e^{-(t-\tau)}u(\tau)u(t-\tau)d\tau\dots\\ ~&~-\int_{-\infty}^{\infty}e^{-(t-\tau)}u(\tau-1)u(t-\tau)d\tau\dots\\ ~&~-\int_{-\infty}^{\infty}e^{-(t-\tau)}u(\tau-3)u(t-\tau)d\tau\\ \end{align} \)

Taking the first part alone:

\( \int_{-\infty}^{\infty}{\color{Brown}2e^{-(t-\tau)}}{\color{Red}u(\tau)}{\color{Blue}u(t-\tau)}d\tau \)

note that the integrand is only non-zero when two conditions are met simultaneously:

\( \begin{align} {\color{Red}0}&{\color{Red}<\tau} & {\color{Blue}\tau}&{\color{Blue}<t} \end{align} \)

Furthermore, those conditions can only be met simultaneously if the outer conditions are met; that is, if:

\( \begin{align} {\color{Purple}0}&{\color{Purple}<t} \end{align} \)

These three conditions lead to three changes. The limits on \(\tau\) change the limits of the integral with respect to \(\tau\) while the condition on \(t\) can be represented by a unit step of \(t\). In other words: